Download Corrections

Survey
yes no Was this document useful for you?
   Thank you for your participation!

* Your assessment is very important for improving the workof artificial intelligence, which forms the content of this project

Document related concepts

Patient safety wikipedia , lookup

Prenatal testing wikipedia , lookup

List of medical mnemonics wikipedia , lookup

Transcript
QUESTION 1:
CORRECT (You correctly answered A)
J.S. is a 70-year old African American male (5'8", 85 kg) with a history of hypertension and CKD. His serum creatinine
today is 1.50 mg/dL (using the IDMS calibrated assay). What is his estimated creatinine clearance?
A. 55.0 mL/min
B. 43.5 mL/min
C. 37.2 mL/min
D. 29.4 mL/min
QUESTION 2:
CORRECT (You correctly answered A)
The decreased serum creatinine values observed during dobutamine therapy are likely due to:
A. analytical interference
B. increased tubular secretion of creatinine
C. increased GFR caused by dobutamine
D. increased muscle breakdown
QUESTION 3:
INCORRECT (You answered A; the correct answer is C)
The kidney is responsible for synthesizing each of the following hormones, EXCEPT:
A. Erythropoietin
B. Prostaglandin
C. PTH
D. Renin
QUESTION 4:
INCORRECT (You answered A; the correct answer is B)
In the clinical setting, the renal clearance of PAH is considered an index of _______.
A. Fractional excretion of sodium
B. Renal plasma or blood flow
C. Glomerular filtration rate
D. Renal tubular reabsorption
QUESTION 5:
CORRECT (You correctly answered A)
The glomerulus is primarily responsible for ______ of unbound drug in the kidney:
A. Filtration
B. Reabsorption
C. Secretion
D. Endocytosis
QUESTION 6:
INCORRECT (You answered A; the correct answer is C)
An appropriate clinical monitoring plan to evaluate renal protective therapy in patients with CKD should include each
of the following items EXCEPT:
A. estimated creatinine clearance
B. urinary albumin:creatinine
C. urinary Cystatin C
D. estimated GFR
QUESTION 7:
INCORRECT (You answered A; the correct answer is B)
Which of the following compounds is most often used to assess cationic tubular secretion?
A. PAH
B. TEA
C. Probenecid
D. Retinal-binding protein
QUESTION 8:
INCORRECT (You answered A; the correct answer is D)
The least accurate method for measuring GFR is:
A. Iohexol clearance
B. Iothalamate clearance
C. Inulin clearance
D. Creatinine clearance
QUESTION 9:
INCORRECT (You answered A; the correct answer is D)
Which of the following renal function indices is least affected by dietary protein intake:
A. Serum creatinine
B. Blood urea nitrogen
C. Creatinine clearance
D. Urine sodium
QUESTION 10:
INCORRECT (You answered A; the correct answer is B)
J.R. is a 68 year-old Caucasian man (60 kg, 5'7") with a history of hypertension, cerebral stroke and benign prostatic
hypertrophy. He presents to the ambulatory care clinic today for evaluation of a viral infection to be treated with
acyclovir. His serum creatinine value today is 0.63 mg/dL. Which one of the following approaches should be used to
assess this patient's renal function for the purpose of renal dose adjustment for acyclovir?
A. Measure a chromium-labeled ethylenediaminetetraacetic acid GFR.
B. Estimate creatinine clearance using the CG equation.
C. Estimate GFR using the MDRD equation.
D. Conduct a timed 24-hour urine collection.
QUESTION 11:
CORRECT (You correctly answered A)
J.S. is a 70-year old African American male (5'8", 85 kg) with a history of hypertension and CKD. His serum creatinine
today is 1.50 mg/dL (using the IDMS calibrated assay). What is J.S.'s estimated GFR when corrected for BSA (in
mL/min)?
A. 38.9 mL/min
B. 45.6 mL/min
C. 65.2 mL/min
D. 72.1 mL/min
QUESTION 12:
INCORRECT (You answered A; the correct answer is D)
J.S. is a 70-year old African American male (5'8", 85 kg) with a history of hypertension and CKD. His serum creatinine
today is 1.50 mg/dL (using the IDMS calibrated assay). What is J.S.'s estimated GFR (in mL/min/1.73m2)?
A. 56.0 mL/min/1.73m2
B. 49.4 mL/min/1.73m2
C. 35.0 mL/min/1.73m2
D. 30.2 mL/min/1.73m2
QUESTION 13:
CORRECT (You correctly answered A)
Which of the following renal function indices is least influenced to changes in fluid or volume status:
A. Serum creatinine
B. Blood urea nitrogen
C. Urine specific gravity
D. Urine sodium
QUESTION 14:
INCORRECT (You answered A; the correct answer is C)
The most appropriate index for quantifying proteinuria in a patient with CKD risk factors is:
A. Total protein dipstick
B. Protein:albumin ratio
C. Albumin:creatinine ratio
D. 24-hour protein excretion
QUESTION 15:
CORRECT (You correctly answered A)
Which of the following equations is most appropriate for estimating renal function in a patient with stable CKD and
GFR < 60 mL/min/1.73m2?
A. 4-variable MDRD
B. 6-variable MDRD
C. Cockcroft-Gault
D. Brater
QUESTION 16:
INCORRECT (You answered A; the correct answer is B)
Active drug secretion occurs most often in which of the following nephron segments:
A. Glomerulus
B. Proximal tubule
C. Loop of Henle
D. Distal tubule
QUESTION 17:
INCORRECT (You answered A; the correct answer is B)
Use of MDRD equations in individuals with GFR > 60 mL/min/1.73m2 may result in: ____________.
A. Overestimation of true GFR
B. Underestimation of true GFR
C. Accurate estimation of true GFR
D. Accurate estimation of creatinine clearance
QUESTION 18:
INCORRECT (You answered A; the correct answer is B)
Which of the following parameters are used to determine the stage of severity of a patient's AKI by means of Risk,
Injury, Failure, Loss of Kidney Function, and End-Stage Renal Disease (RIFLE) and Acute Kidney Injury Network
(AKIN) classification systems?
A. Serum creatinine and blood urea nitrogen
B. Serum creatinine and urine output
C. Glomerular filtration rate and blood urea nitrogen
D. Glomerular filtration rate and cystatin C
QUESTION 19:
INCORRECT (You answered A; the correct answer is D)
According to the intact nephron hypothesis, reabsorption ______ and single nephron GFR ______ in the surviving
nephrons:
A. increases, increases
B. decreases, decreases
C. increases, decreases
D. decreases, increases
QUESTION 20:
CORRECT (You correctly answered A)
A 60-year-old long-term care resident is admitted to the hospital with altered mental status. His admission laboratory
values show a blood urea nitrogen (BUN) of 30 mg/dL (10.7 mmol/L), serum creatinine (Scr) of 2 mg/dL (177
µmol/L), a fractional excretion of sodium (FeNa) of 2.5% (0.025), and granular casts on urine sediment. The most
likely etiology of his AKI is
A. Acute tubular necrosis
B. Bladder obstruction
C. Nonsteroidal antiinflammatory drug (NSAID)–induced renal hypoperfusion
D. Volume depletion
QUESTION 21:
INCORRECT (You answered A; the correct answer is D)
Which of the following combinations would be most effective to enhance urine production in a patient who has oliguric
ARF secondary to ATN?
A. Furosemide and ethacrynic acid
B. Triamterene and hydrochlorothiazide
C. Bumetanide and spironolactone
D. Furosemide and metolazone
E. Spironolactone and metolazone
QUESTION 22:
INCORRECT (You answered A; the correct answer is C)
A 56-year-old (71 kg) man is admitted to the intensive care unit with sepsis. His serum creatinine increased from a
baseline of 0.9 mg/dL (80 µmol/L) to 1.6 mg/dL (141 µmol/L), and his blood urea nitrogen (BUN) increased from 15
mg/dL (5.4 mmol/L) to 30 mg/dL (10.7 mmol/L). His urine output in the last 24 hours was 500 mL. Per RIFLE criteria,
which stage does this patient's AKI belong to?
A. Risk
B. Injury
C. Failure
D. Loss of kidney function
E. End-stage kidney disease
QUESTION 23:
INCORRECT (You answered A; the correct answer is D)
All of the following factors can make drug dosing a challenge in a critically ill patient with established AKI except the
A. Presence of edema, which can significantly increase the volume of distribution of a drug
B. Presence of residual nonrenal clearance
C. Need for constant reassessment of the patient's renal function and status
D. Presence of electrolyte abnormalities
QUESTION 24:
INCORRECT (You answered A; the correct answer is B)
Presentation
CC: Primary care physician referral
HPI: An 83-year-old woman is referred to the hospital for a renal workup. The patient had complaints of
decreased urine output (UO) and a 10-lb weight gain 1 week PTA according to her daughter. On examination
by the medical housestaff, the patient denies chest pain, palpitations, dyspnea, or abdominal pain.
PMH: Congestive heart failure (CHF), hypertension (HTN), urinary incontinence, gout (no active medications),
mild dementia, moderate renal insufficiency (baseline serum creatinine 1.3)
Medications on admission: Nitroglycerin (10 mg) transdermal 0.4 g/h qam, ramipril (Altace) 2.5 mg qd,
hydrochlorothiazide (Hydrodiuril) 25 mg qd, multivitamin 1 tab qd, oxybutynin (Ditropan) 5 mg qd
ALL: Penicillin
FH: Not available
SH: Patient lives with daughter; no alcohol use; 30 pack-year history of tobacco use
PE: Patient looks younger than stated age, lying in bed, comfortable and in NAD.
ROS: Significant findings: MMM, cardiac murmur, 2(+) pedal edema BL, A&O x 2
VS: Temperature: 98.2°F; heart rate: 90 beats per minute; blood pressure: 140/90 mm Hg; respiratory rate:
18 breaths per minute; O2 saturation: 97% RA
Weight: 129 lb (baseline 119 lb); height: 5 ft, 2 in
Laboratory Values
Na: 130
Cl: 107 Total
Blood urea nitrogen (BUN): 82
BG: 88
Hct: 30.4%
Platelets: 212
K: 6.4
CO2: 18
Serum Cr: 4.0
White blood cell (WBC) count: 10.1
Hgb: 10.0
U/A: C&S pending; SG. 1.015; pH 7.5, Na: 20, Cr: 52, /hpf RBC: 0–2; WBC: 6–10,
Bacteria (occ); yeast (–)
UO: < 10 mL/h
ECG: No QRS widening or T-wave elevations
Chest x-ray: Bilateral pleural effusions; no evidence of pneumonia
Medications administered in emergency room: Regular insulin 10 units and 50% dextrose 50 mL IV x 1,
ciprofloxacin (Cipro) 400 mg IV q12h
Recommend the most appropriate agent in this case to convert the patient from oliguria to nonoliguria.
A. Nesiritide IV
B. Furosemide IV
C. Metolazone PO
D. Chorothiazide IV
E. Hydrochlorothiazide PO
QUESTION 25:
INCORRECT (You answered A; the correct answer is C)
A patient in the intensive care unit develops AKI. You review the medications the patient has been taking to evaluate
for drug-induced AKI. Which of the following agents would be most likely to cause AIN?
A. Labetalol
B. Diltiazem
C. Cefazolin
D. Fentanyl
E. Propofol
QUESTION 26:
INCORRECT (You answered A; the correct answer is B)
Which of the following electrolyte abnormalities are most commonly found in patients with AKI?
A. Hypophosphatemia
B. Hyperkalemia
C. Hyponatremia
D. None of the above; electrolytes are usually unaffected
QUESTION 27:
INCORRECT (You answered A; the correct answer is D)
Most patients suffer from acute renal failure as a result of
A. dehydration
B. nitrosis
C. cirrhosis
D. bacterial infection
E. hyperkalemia
QUESTION 28:
INCORRECT (You answered A; the correct answer is E)
Presentation
CC: Patient was transported from home to the emergency room in a private ambulance.
HPI: The patient is a 67-year-old man who was discharged from the hospital with resolving pneumonia one
day PTA. He is accompanied by his son, who claims to have found him unconscious on the bathroom floor. On
arrival to the emergency room, the patient remains unconscious and unresponsive to stimuli, with shallow
respiratory breathing.
PMH: Type 2 diabetes mellitus (DM) x 15 years, hypertension (HTN) x 10 years, mild chronic renal
insufficiency (CRI) x 2 years, congestive heart failure (ACC/AHA stage B) x 1 year, asthma (unknown history),
community-acquired pneumonia (recent hospital admission), headaches
Medications on admission: Levofloxacin (Levoquin) 500 mg PO qd x 1 week left, lisinopril (Zestril) 20 mg qd,
furosemide (Lasix) 20 mg qd in the morning, carvedilol (Coreg) 25 mg bid, glyburide (Micronase) 5 mg qd,
albuterol (Proventil) IH 2 puffs qid PRN, ibuprofen (Advil) 200 mg (2 tablets) PRN for headaches
ALL: NKDA
SH: Widower, lives alone, retired
FH: Noncontributory
Physical Examination
General: White male who is semiconscious and ill-appearing with labored breathing.
HEENT: EOMI, PERRLA, MMM
Neck: Soft, supple
COR: RRR, S1, S2 gallop
PULM: Rales lower left lobe
ABD: Soft, nontender, nondistended
EXT: Positive distal pulses, positive BL edema
Neuro: Not able to perform tests
VS: Temperature: 98.7°F; blood pressure: 124/84 mm Hg; heart rate: 80 beats per minute; respiratory rate:
15 breaths per minute
Weight: 58 kg; height: 5 ft, 6 in
Laboratory Values
Na: 140 mEq/L
Cl: 99
Blood urea nitrogen (BUN): 101
Glucose: 160
Red blood cell (RBC) count: 4.2
Hgb: 11.4
Ca: 7.9
Albumin: 4.0
U/A: No sediment, no protein, Gram's stain (–)
Urine output: <10 mL/h
ECG: Flattened P waves, widened QRS segment
K: 7.5
Total CO2: 18
Serum Cr: 6.2 (baseline 1.4)
White blood cell (WBC) count: 15
Hct: 35%
Platelets: 180
PO4: 8.2
The medical team makes the following assessments:
1. ECG changes consistent with life-threatening hyperkalemia
2. Oliguric acute renal failure probably due prerenal azotemia from volume overload
3. r/o septicemia
Which of the problems listed below is(are) biochemical and hemodynamic manifestations consistent with acute renal
failure (ARF)?
I. Metabolic acidosis
II. Hyperkalemia
III. Oliguria
A. I only
B. III only
C. I and II only
D. II and III only
E. I, II, and III
QUESTION 29:
INCORRECT (You answered D; the correct answer is B)
Presentation
CC: Primary care physician referral
HPI: An 83-year-old woman is referred to the hospital for a renal workup. The patient had complaints of
decreased urine output (UO) and a 10-lb weight gain 1 week PTA according to her daughter. On examination
by the medical housestaff, the patient denies chest pain, palpitations, dyspnea, or abdominal pain.
PMH: Congestive heart failure (CHF), hypertension (HTN), urinary incontinence, gout (no active medications),
mild dementia, moderate renal insufficiency (baseline serum creatinine 1.3)
Medications on admission: Nitroglycerin (10 mg) transdermal 0.4 g/h qam, ramipril (Altace) 2.5 mg qd,
hydrochlorothiazide (Hydrodiuril) 25 mg qd, multivitamin 1 tab qd, oxybutynin (Ditropan) 5 mg qd
ALL: Penicillin
FH: Not available
SH: Patient lives with daughter; no alcohol use; 30 pack-year history of tobacco use
PE: Patient looks younger than stated age, lying in bed, comfortable and in NAD.
ROS: Significant findings: MMM, cardiac murmur, 2(+) pedal edema BL, A&O x 2
VS: Temperature: 98.2°F; heart rate: 90 beats per minute; blood pressure: 140/90 mm Hg; respiratory rate:
18 breaths per minute; O2 saturation: 97% RA
Weight: 129 lb (baseline 119 lb); height: 5 ft, 2 in
Laboratory Values
Na: 130
Cl: 107 Total
Blood urea nitrogen (BUN): 82
BG: 88
Hct: 30.4%
Platelets: 212
K: 6.4
CO2: 18
Serum Cr: 4.0
White blood cell (WBC) count: 10.1
Hgb: 10.0
U/A: C&S pending; SG. 1.015; pH 7.5, Na: 20, Cr: 52, /hpf RBC: 0–2; WBC: 6–10,
Bacteria (occ); yeast (–)
UO: < 10 mL/h
ECG: No QRS widening or T-wave elevations
Chest x-ray: Bilateral pleural effusions; no evidence of pneumonia
Medications administered in emergency room: Regular insulin 10 units and 50% dextrose 50 mL IV x 1,
ciprofloxacin (Cipro) 400 mg IV q12h
Choose the first or highest-priority problem on the list from the following assessments for this case.
A. Hyperkalemia most likely owing to acute renal failure (ARF) with no significant ECG changes
B. Oliguric ARF probably owing to cardiac decompensation, thiazide diuretic or ACE inhibitor use, or medication
noncompliance
C. Decreased total CO2 may indicate non–life-threatening metabolic acidosis.
D. Elevated blood pressure most likely owing to volume overload
E. Hyponatremia most likely owing to fluid overload: 2(+) pedal edema BL; 10-lb weight gain 1 week PTA
QUESTION 30:
INCORRECT (You answered D; the correct answer is E)
Presentation
CC: Primary care physician referral
HPI: An 83-year-old woman is referred to the hospital for a renal workup. The patient had complaints of
decreased urine output (UO) and a 10-lb weight gain 1 week PTA according to her daughter. On examination
by the medical housestaff, the patient denies chest pain, palpitations, dyspnea, or abdominal pain.
PMH: Congestive heart failure (CHF), hypertension (HTN), urinary incontinence, gout (no active medications),
mild dementia, moderate renal insufficiency (baseline serum creatinine 1.3)
Medications on admission: Nitroglycerin (10 mg) transdermal 0.4 g/h qam, ramipril (Altace) 2.5 mg qd,
hydrochlorothiazide (Hydrodiuril) 25 mg qd, multivitamin 1 tab qd, oxybutynin (Ditropan) 5 mg qd
ALL: Penicillin
FH: Not available
SH: Patient lives with daughter; no alcohol use; 30 pack-year history of tobacco use
PE: Patient looks younger than stated age, lying in bed, comfortable and in NAD.
ROS: Significant findings: MMM, cardiac murmur, 2(+) pedal edema BL, A&O x 2
VS: Temperature: 98.2°F; heart rate: 90 beats per minute; blood pressure: 140/90 mm Hg; respiratory rate:
18 breaths per minute; O2 saturation: 97% RA
Weight: 129 lb (baseline 119 lb); height: 5 ft, 2 in
Laboratory Values
Na: 130
Cl: 107 Total
Blood urea nitrogen (BUN): 82
BG: 88
Hct: 30.4%
Platelets: 212
K: 6.4
CO2: 18
Serum Cr: 4.0
White blood cell (WBC) count: 10.1
Hgb: 10.0
U/A: C&S pending; SG. 1.015; pH 7.5, Na: 20, Cr: 52, /hpf RBC: 0–2; WBC: 6–10,
Bacteria (occ); yeast (–)
UO: < 10 mL/h
ECG: No QRS widening or T-wave elevations
Chest x-ray: Bilateral pleural effusions; no evidence of pneumonia
Medications administered in emergency room: Regular insulin 10 units and 50% dextrose 50 mL IV x 1,
ciprofloxacin (Cipro) 400 mg IV q12h
All the following statements are true regarding medication use in renal failure pertaining to this patient except
A. Thiazide diuretics such as hydrochlorothiazide worsen renal failure in the presence of ClCr < 30 mL/min.
B. The dose of fluoroquinolones such as ciprofloxacin should be reduced in the presence of renal failure to avoid
nephrotoxicity.
C. Angiotensin-converting enzyme (ACE) inhibitors such as ramipril should be discontinued temporarily in the
presence of renal failure and hyperkalemia.
D. Combination therapy with two different diuretics may be successful in patients resistant to a single agent.
E. The calcium channel blocker nifedipine should be added to the antihypertensive regimen.
QUESTION 31:
INCORRECT (You answered A; the correct answer is D)
Which of the following pathophysiologic processes are involved in the development of prerenal AKI?
A. Glomerular damage secondary to severe inflammation
B. Tubular epithelial cell necrosis due to ischemia
C. Drug hypersensitivity reaction leading to interstitial inflammation
D. Decreased renal perfusion secondary to volume depletion
QUESTION 32:
INCORRECT (You answered A; the correct answer is D)
Which of the following statements is true regarding drug dosing in AKI?
A. All patients should be dosed for a creatinine clearance <10 mL/min.
B. Pharmacokinetic parameters do not usually change so dose adjustment is not needed.
C. Although elimination may be decreased, the volume of distribution should remain unchanged in AKI.
D. The estimation of kidney function should include urine output.
E. All drugs should be monitored by serum levels to ensure safety and efficacy.
QUESTION 33:
INCORRECT (You answered A; the correct answer is B)
Which of the following parameters are used to determine the stage of severity of a patient's AKI by means of Risk,
Injury, Failure, Loss of Kidney Function, and End-Stage Renal Disease (RIFLE) and Acute Kidney Injury Network
(AKIN) classification systems?
A. Serum creatinine and blood urea nitrogen
B. Serum creatinine and urine output
C. Glomerular filtration rate and blood urea nitrogen
D. Glomerular filtration rate and cystatin C
QUESTION 34:
CORRECT (You correctly answered B)
Presentation
CC: Patient was transported from home to the emergency room in a private ambulance.
HPI: The patient is a 67-year-old man who was discharged from the hospital with resolving pneumonia one
day PTA. He is accompanied by his son, who claims to have found him unconscious on the bathroom floor. On
arrival to the emergency room, the patient remains unconscious and unresponsive to stimuli, with shallow
respiratory breathing.
PMH: Type 2 diabetes mellitus (DM) x 15 years, hypertension (HTN) x 10 years, mild chronic renal
insufficiency (CRI) x 2 years, congestive heart failure (ACC/AHA stage B) x 1 year, asthma (unknown history),
community-acquired pneumonia (recent hospital admission), headaches
Medications on admission: Levofloxacin (Levoquin) 500 mg PO qd x 1 week left, lisinopril (Zestril) 20 mg qd,
furosemide (Lasix) 20 mg qd in the morning, carvedilol (Coreg) 25 mg bid, glyburide (Micronase) 5 mg qd,
albuterol (Proventil) IH 2 puffs qid PRN, ibuprofen (Advil) 200 mg (2 tablets) PRN for headaches
ALL: NKDA
SH: Widower, lives alone, retired
FH: Noncontributory
Physical Examination
General: White male who is semiconscious and ill-appearing with labored breathing.
HEENT: EOMI, PERRLA, MMM
Neck: Soft, supple
COR: RRR, S1, S2 gallop
PULM: Rales lower left lobe
ABD: Soft, nontender, nondistended
EXT: Positive distal pulses, positive BL edema
Neuro: Not able to perform tests
VS: Temperature: 98.7°F; blood pressure: 124/84 mm Hg; heart rate: 80 beats per minute; respiratory rate:
15 breaths per minute
Weight: 58 kg; height: 5 ft, 6 in
Laboratory Values
Na: 140 mEq/L
Cl: 99
Blood urea nitrogen (BUN): 101
Glucose: 160
Red blood cell (RBC) count: 4.2
Hgb: 11.4
Ca: 7.9
Albumin: 4.0
U/A: No sediment, no protein, Gram's stain (–)
Urine output: <10 mL/h
ECG: Flattened P waves, widened QRS segment
K: 7.5
Total CO2: 18
Serum Cr: 6.2 (baseline 1.4)
White blood cell (WBC) count: 15
Hct: 35%
Platelets: 180
PO4: 8.2
The medical team makes the following assessments:
1. ECG changes consistent with life-threatening hyperkalemia
2. Oliguric acute renal failure probably due prerenal azotemia from volume overload
3. r/o septicemia
Choose the diuretic of first choice for converting the patient from oliguria to nonoliguria?
A. Spironolactone
B. Furosemide
C. Hydrochlorothiazide
D. Metolazone
E. Mannitol
QUESTION 35:
CORRECT (You correctly answered A)
Which of the following medications are associated with the development of prerenal AKI?
A. Valsartan
B. Acyclovir
C. Lithium
D. Gentamicin
QUESTION 36:
INCORRECT (You answered A; the correct answer is D)
Which of the following pharmacotherapeutic interventions is used to reverse AKI in hospitalized patients?
A. Saline hydration
B. Intermittent hemodialysis
C. Continuous renal replacement therapy
D. None of the above; supportive care is the mainstay of therapy
QUESTION 37:
INCORRECT (You answered A; the correct answer is B)
PH is a 68-year-old man who was admitted to the hospital yesterday with an acute myocardial infarction. The
cardiologists want to perform a cardiac catheterization with radio contrast dye. Which of the following regimens would
be best as prophylaxis for contrast-induced nephropathy in this patient?
A. N-acetylcysteine 150 mg/kg IV for 6 hours preprocedure.
B. Sodium chloride 0.9% IV infusion 6 hours before and 6 hours after.
C. Theophylline 200 mg po every 12 hours, two doses before, two doses after.
D. Dopamine 0.5
g/kg/min IV infusion 6 hours before and 6 hours after.
E. No prophylaxis is recommended.
QUESTION 38:
INCORRECT (You answered A; the correct answer is B)
Which of the following statements is true regarding the blood urea nitrogen (BUN) to serum creatinine (SCr) ratio?
A. In situations of dehydration the BUN:SCr will be <10.
B. In situations of dehydration the BUN:SCr will be >15.
C. In situations of volume overload the BUN:SCr will be <10.
D. In situations of volume overload the BUN:SCr will be >15.
E. In situations of glomerulonephritis the BUN:SCr will be <5.
QUESTION 39:
INCORRECT (You answered A; the correct answer is D)
Which of the following agents can cause constriction of the afferent arteriole?
A. Nonsteroidal anti-inflammatory agents (NSAIDs)
B. Calcineurin inhibitors
C. Angiotensin-converting enzyme inhibitors (ACE-I)
D. a and b only
E. a, b, and c
QUESTION 40:
CORRECT (You correctly answered A)
Compared with intermittent hemodialysis, one of the main advantages of continuous renal replacement therapy is that
A. It is associated with fewer hypotensive episodes
B. It is associated with lower rates of thrombosis
C. It is less labor intensive
D. It does not require anticoagulation
QUESTION 41:
CORRECT (You correctly answered B)
Presentation
CC: Primary care physician referral
HPI: An 83-year-old woman is referred to the hospital for a renal workup. The patient had complaints of
decreased urine output (UO) and a 10-lb weight gain 1 week PTA according to her daughter. On examination
by the medical housestaff, the patient denies chest pain, palpitations, dyspnea, or abdominal pain.
PMH: Congestive heart failure (CHF), hypertension (HTN), urinary incontinence, gout (no active medications),
mild dementia, moderate renal insufficiency (baseline serum creatinine 1.3)
Medications on admission: Nitroglycerin (10 mg) transdermal 0.4 g/h qam, ramipril (Altace) 2.5 mg qd,
hydrochlorothiazide (Hydrodiuril) 25 mg qd, multivitamin 1 tab qd, oxybutynin (Ditropan) 5 mg qd
ALL: Penicillin
FH: Not available
SH: Patient lives with daughter; no alcohol use; 30 pack-year history of tobacco use
PE: Patient looks younger than stated age, lying in bed, comfortable and in NAD.
ROS: Significant findings: MMM, cardiac murmur, 2(+) pedal edema BL, A&O x 2
VS: Temperature: 98.2°F; heart rate: 90 beats per minute; blood pressure: 140/90 mm Hg; respiratory rate:
18 breaths per minute; O2 saturation: 97% RA
Weight: 129 lb (baseline 119 lb); height: 5 ft, 2 in
Laboratory Values
Na: 130
Cl: 107 Total
Blood urea nitrogen (BUN): 82
BG: 88
Hct: 30.4%
Platelets: 212
K: 6.4
CO2: 18
Serum Cr: 4.0
White blood cell (WBC) count: 10.1
Hgb: 10.0
U/A: C&S pending; SG. 1.015; pH 7.5, Na: 20, Cr: 52, /hpf RBC: 0–2; WBC: 6–10,
Bacteria (occ); yeast (–)
UO: < 10 mL/h
ECG: No QRS widening or T-wave elevations
Chest x-ray: Bilateral pleural effusions; no evidence of pneumonia
Medications administered in emergency room: Regular insulin 10 units and 50% dextrose 50 mL IV x 1,
ciprofloxacin (Cipro) 400 mg IV q12h
What is the most likely indication for combination use of a loop and thiazide diuretic regimen in this patient?
A. Hyperuricemia
B. Diuretic resistance
C. Hyponatremia
D. Nephrotic syndrome
E. Hyperkalemia
QUESTION 42:
INCORRECT (You answered A; the correct answer is B)
A 66-year-old woman with a history of diabetes and chronic kidney disease is scheduled for diagnostic imaging
requiring contrast dye administration. Her serum creatinine is 2 mg/dL (177 µmol/L), blood urea nitrogen (BUN) is 30
mg/dL (10.7 mmoL/L), and urine output in the last 24 hours is 1,500 mL. Her complete blood count and electrolytes
are all within normal range. Which of the following medications would you recommend to decrease the risk of
contrast-induced nephropathy in this patient?
A. Tight glycemic control
B. Sodium bicarbonate infusion
C. Hemodialysis
D. Furosemide infusion
E. None of the abov
QUESTION 43: INCORRECT (You answered D; the correct answer is E)
Presentation
CC: Primary care physician referral
HPI: An 83-year-old woman is referred to the hospital for a renal workup. The patient had complaints of
decreased urine output (UO) and a 10-lb weight gain 1 week PTA according to her daughter. On examination
by the medical housestaff, the patient denies chest pain, palpitations, dyspnea, or abdominal pain.
PMH: Congestive heart failure (CHF), hypertension (HTN), urinary incontinence, gout (no active medications),
mild dementia, moderate renal insufficiency (baseline serum creatinine 1.3)
Medications on admission: Nitroglycerin (10 mg) transdermal 0.4 g/h qam, ramipril (Altace) 2.5 mg qd,
hydrochlorothiazide (Hydrodiuril) 25 mg qd, multivitamin 1 tab qd, oxybutynin (Ditropan) 5 mg qd
ALL: Penicillin
FH: Not available
SH: Patient lives with daughter; no alcohol use; 30 pack-year history of tobacco use
PE: Patient looks younger than stated age, lying in bed, comfortable and in NAD.
ROS: Significant findings: MMM, cardiac murmur, 2(+) pedal edema BL, A&O x 2
VS: Temperature: 98.2°F; heart rate: 90 beats per minute; blood pressure: 140/90 mm Hg; respiratory rate:
18 breaths per minute; O2 saturation: 97% RA
Weight: 129 lb (baseline 119 lb); height: 5 ft, 2 in
Laboratory Values
Na: 130
Cl: 107 Total
Blood urea nitrogen (BUN): 82
BG: 88
Hct: 30.4%
Platelets: 212
K: 6.4
CO2: 18
Serum Cr: 4.0
White blood cell (WBC) count: 10.1
Hgb: 10.0
U/A: C&S pending; SG. 1.015; pH 7.5, Na: 20, Cr: 52, /hpf RBC: 0–2; WBC: 6–10,
Bacteria (occ); yeast (–)
UO: < 10 mL/h
ECG: No QRS widening or T-wave elevations
Chest x-ray: Bilateral pleural effusions; no evidence of pneumonia
Medications administered in emergency room: Regular insulin 10 units and 50% dextrose 50 mL IV x 1,
ciprofloxacin (Cipro) 400 mg IV q12h
The patient remains oliguric on hospital day 2 despite aggressive diuretic therapy with adequate IV hydration to avoid
volume depletion. Serum creatinine and BUN are still elevated at 3.0 and 61, respectively. Serum K + is 5.0. Provide a
plausible explanation for worsening renal perfusion in this patient.
A. Higher than usual doses of diuretics are necessary in the presence of ARF.
B. Worsening CHF
C. Volume overload
D. Hypoalbuminemia
E. All the above
QUESTION 44:
INCORRECT (You answered E; the correct answer is C)
Presentation
CC: Primary care physician referral
HPI: An 83-year-old woman is referred to the hospital for a renal workup. The patient had complaints of
decreased urine output (UO) and a 10-lb weight gain 1 week PTA according to her daughter. On examination
by the medical housestaff, the patient denies chest pain, palpitations, dyspnea, or abdominal pain.
PMH: Congestive heart failure (CHF), hypertension (HTN), urinary incontinence, gout (no active medications),
mild dementia, moderate renal insufficiency (baseline serum creatinine 1.3)
Medications on admission: Nitroglycerin (10 mg) transdermal 0.4 g/h qam, ramipril (Altace) 2.5 mg qd,
hydrochlorothiazide (Hydrodiuril) 25 mg qd, multivitamin 1 tab qd, oxybutynin (Ditropan) 5 mg qd
ALL: Penicillin
FH: Not available
SH: Patient lives with daughter; no alcohol use; 30 pack-year history of tobacco use
PE: Patient looks younger than stated age, lying in bed, comfortable and in NAD.
ROS: Significant findings: MMM, cardiac murmur, 2(+) pedal edema BL, A&O x 2
VS: Temperature: 98.2°F; heart rate: 90 beats per minute; blood pressure: 140/90 mm Hg; respiratory rate:
18 breaths per minute; O2 saturation: 97% RA
Weight: 129 lb (baseline 119 lb); height: 5 ft, 2 in
Laboratory Values
Na: 130
Cl: 107 Total
Blood urea nitrogen (BUN): 82
BG: 88
Hct: 30.4%
Platelets: 212
K: 6.4
CO2: 18
Serum Cr: 4.0
White blood cell (WBC) count: 10.1
Hgb: 10.0
U/A: C&S pending; SG. 1.015; pH 7.5, Na: 20, Cr: 52, /hpf RBC: 0–2; WBC: 6–10,
Bacteria (occ); yeast (–)
UO: < 10 mL/h
ECG: No QRS widening or T-wave elevations
Chest x-ray: Bilateral pleural effusions; no evidence of pneumonia
Medications administered in emergency room: Regular insulin 10 units and 50% dextrose 50 mL IV x 1,
ciprofloxacin (Cipro) 400 mg IV q12h
The patient's calculated FeNa is 1.8%. All the following statements are true except
A. FeNa helps to establish the diagnosis of ARF.
B. Prerenal azotemia may be differentiated from intrinsic renal failure when the FeNa value is < 1%.
C. Prerenal azotemia may be differentiated from intrinsic renal failure when the FeNa value is
1%.
D. Confounding variables such as diuretic use may influence the calculated FeNa, leading to a false-positive value.
E. Both urinary and serum sodium and creatinine values are needed to calculate FeNa.
QUESTION 45:
CORRECT (You correctly answered D)
Presentation
CC: Primary care physician referral
HPI: An 83-year-old woman is referred to the hospital for a renal workup. The patient had complaints of
decreased urine output (UO) and a 10-lb weight gain 1 week PTA according to her daughter. On examination
by the medical housestaff, the patient denies chest pain, palpitations, dyspnea, or abdominal pain.
PMH: Congestive heart failure (CHF), hypertension (HTN), urinary incontinence, gout (no active medications),
mild dementia, moderate renal insufficiency (baseline serum creatinine 1.3)
Medications on admission: Nitroglycerin (10 mg) transdermal 0.4 g/h qam, ramipril (Altace) 2.5 mg qd,
hydrochlorothiazide (Hydrodiuril) 25 mg qd, multivitamin 1 tab qd, oxybutynin (Ditropan) 5 mg qd
ALL: Penicillin
FH: Not available
SH: Patient lives with daughter; no alcohol use; 30 pack-year history of tobacco use
PE: Patient looks younger than stated age, lying in bed, comfortable and in NAD.
ROS: Significant findings: MMM, cardiac murmur, 2(+) pedal edema BL, A&O x 2
VS: Temperature: 98.2°F; heart rate: 90 beats per minute; blood pressure: 140/90 mm Hg; respiratory rate:
18 breaths per minute; O2 saturation: 97% RA
Weight: 129 lb (baseline 119 lb); height: 5 ft, 2 in
Laboratory Values
Na: 130
Cl: 107 Total
Blood urea nitrogen (BUN): 82
BG: 88
Hct: 30.4%
Platelets: 212
K: 6.4
CO2: 18
Serum Cr: 4.0
White blood cell (WBC) count: 10.1
Hgb: 10.0
U/A: C&S pending; SG. 1.015; pH 7.5, Na: 20, Cr: 52, /hpf RBC: 0–2; WBC: 6–10,
Bacteria (occ); yeast (–)
UO: < 10 mL/h
ECG: No QRS widening or T-wave elevations
Chest x-ray: Bilateral pleural effusions; no evidence of pneumonia
Medications administered in emergency room: Regular insulin 10 units and 50% dextrose 50 mL IV x 1,
ciprofloxacin (Cipro) 400 mg IV q12h
Identify the most important missing patient-specific information needed to properly evaluate, treat, and monitor renal
insufficiency.
A. Specific allergy history
B. Medication compliance, including over-the- counter (OTC) and herbals medications, Na, and dietary and fluid
intake
C. At home blood pressure measurements, baseline mental status
D. Baseline serum creatinine, BUN, EF, arterial blood gases (ABGs), blood pH
E. Urine C&S results
QUESTION 46:
INCORRECT (You answered B; the correct answer is E)
A 60-year-old woman with a 5-year history of NSAID use is prescribed enalapril and develops acute kidney injury.
What is the most likely cause of her acute kidney injury?
A. Acute allergic interstitial nephritis
B. Chronic interstitial nephritis
C. Minimal change glomerular injury
D. Focal segmental glomerulosclerosis
E. Hemodynamically mediated kidney injury
QUESTION 47:
INCORRECT (You answered A; the correct answer is D)
Presentation
CC: Patient was transported from home to the emergency room in a private ambulance.
HPI: The patient is a 67-year-old man who was discharged from the hospital with resolving pneumonia one
day PTA. He is accompanied by his son, who claims to have found him unconscious on the bathroom floor. On
arrival to the emergency room, the patient remains unconscious and unresponsive to stimuli, with shallow
respiratory breathing.
PMH: Type 2 diabetes mellitus (DM) x 15 years, hypertension (HTN) x 10 years, mild chronic renal
insufficiency (CRI) x 2 years, congestive heart failure (ACC/AHA stage B) x 1 year, asthma (unknown history),
community-acquired pneumonia (recent hospital admission), headaches
Medications on admission: Levofloxacin (Levoquin) 500 mg PO qd x 1 week left, lisinopril (Zestril) 20 mg qd,
furosemide (Lasix) 20 mg qd in the morning, carvedilol (Coreg) 25 mg bid, glyburide (Micronase) 5 mg qd,
albuterol (Proventil) IH 2 puffs qid PRN, ibuprofen (Advil) 200 mg (2 tablets) PRN for headaches
ALL: NKDA
SH: Widower, lives alone, retired
FH: Noncontributory
Physical Examination
General: White male who is semiconscious and ill-appearing with labored breathing.
HEENT: EOMI, PERRLA, MMM
Neck: Soft, supple
COR: RRR, S1, S2 gallop
PULM: Rales lower left lobe
ABD: Soft, nontender, nondistended
EXT: Positive distal pulses, positive BL edema
Neuro: Not able to perform tests
VS: Temperature: 98.7°F; blood pressure: 124/84 mm Hg; heart rate: 80 beats per minute; respiratory rate:
15 breaths per minute
Weight: 58 kg; height: 5 ft, 6 in
Laboratory Values
Na: 140 mEq/L
Cl: 99
Blood urea nitrogen (BUN): 101
Glucose: 160
Red blood cell (RBC) count: 4.2
Hgb: 11.4
Ca: 7.9
Albumin: 4.0
U/A: No sediment, no protein, Gram's stain (–)
Urine output: <10 mL/h
ECG: Flattened P waves, widened QRS segment
K: 7.5
Total CO2: 18
Serum Cr: 6.2 (baseline 1.4)
White blood cell (WBC) count: 15
Hct: 35%
Platelets: 180
PO4: 8.2
The medical team makes the following assessments:
1. ECG changes consistent with life-threatening hyperkalemia
2. Oliguric acute renal failure probably due prerenal azotemia from volume overload
3. r/o septicemia
Which of the following diagnostic tools may be used to differentiate drug-induced prerenal azotemia from intrinsic
renal failure?
A. Urine chemistries
B. Urinalysis
C. Glycosylated hemoglobin A1C
D. A and B
E. A and C
QUESTION 48:
INCORRECT (You answered D; the correct answer is E)
Presentation
CC: Patient was transported from home to the emergency room in a private ambulance.
HPI: The patient is a 67-year-old man who was discharged from the hospital with resolving pneumonia one
day PTA. He is accompanied by his son, who claims to have found him unconscious on the bathroom floor. On
arrival to the emergency room, the patient remains unconscious and unresponsive to stimuli, with shallow
respiratory breathing.
PMH: Type 2 diabetes mellitus (DM) x 15 years, hypertension (HTN) x 10 years, mild chronic renal
insufficiency (CRI) x 2 years, congestive heart failure (ACC/AHA stage B) x 1 year, asthma (unknown history),
community-acquired pneumonia (recent hospital admission), headaches
Medications on admission: Levofloxacin (Levoquin) 500 mg PO qd x 1 week left, lisinopril (Zestril) 20 mg qd,
furosemide (Lasix) 20 mg qd in the morning, carvedilol (Coreg) 25 mg bid, glyburide (Micronase) 5 mg qd,
albuterol (Proventil) IH 2 puffs qid PRN, ibuprofen (Advil) 200 mg (2 tablets) PRN for headaches
ALL: NKDA
SH: Widower, lives alone, retired
FH: Noncontributory
Physical Examination
General: White male who is semiconscious and ill-appearing with labored breathing.
HEENT: EOMI, PERRLA, MMM
Neck: Soft, supple
COR: RRR, S1, S2 gallop
PULM: Rales lower left lobe
ABD: Soft, nontender, nondistended
EXT: Positive distal pulses, positive BL edema
Neuro: Not able to perform tests
VS: Temperature: 98.7°F; blood pressure: 124/84 mm Hg; heart rate: 80 beats per minute; respiratory rate:
15 breaths per minute
Weight: 58 kg; height: 5 ft, 6 in
Laboratory Values
Na: 140 mEq/L
Cl: 99
Blood urea nitrogen (BUN): 101
Glucose: 160
Red blood cell (RBC) count: 4.2
Hgb: 11.4
Ca: 7.9
Albumin: 4.0
U/A: No sediment, no protein, Gram's stain (–)
Urine output: <10 mL/h
ECG: Flattened P waves, widened QRS segment
K: 7.5
Total CO2: 18
Serum Cr: 6.2 (baseline 1.4)
White blood cell (WBC) count: 15
Hct: 35%
Platelets: 180
PO4: 8.2
The medical team makes the following assessments:
1. ECG changes consistent with life-threatening hyperkalemia
2. Oliguric acute renal failure probably due prerenal azotemia from volume overload
3. r/o septicemia
All the following are considered risk factors for ARF-associated mortality except
A. advanced age.
B. oliguria.
C. markedly elevated serum creatinine.
D. septicemia.
E. hyperglycemia.
QUESTION 49:
INCORRECT (You answered D; the correct answer is A)
Presentation
CC: Primary care physician referral
HPI: An 83-year-old woman is referred to the hospital for a renal workup. The patient had complaints of
decreased urine output (UO) and a 10-lb weight gain 1 week PTA according to her daughter. On examination
by the medical housestaff, the patient denies chest pain, palpitations, dyspnea, or abdominal pain.
PMH: Congestive heart failure (CHF), hypertension (HTN), urinary incontinence, gout (no active medications),
mild dementia, moderate renal insufficiency (baseline serum creatinine 1.3)
Medications on admission: Nitroglycerin (10 mg) transdermal 0.4 g/h qam, ramipril (Altace) 2.5 mg qd,
hydrochlorothiazide (Hydrodiuril) 25 mg qd, multivitamin 1 tab qd, oxybutynin (Ditropan) 5 mg qd
ALL: Penicillin
FH: Not available
SH: Patient lives with daughter; no alcohol use; 30 pack-year history of tobacco use
PE: Patient looks younger than stated age, lying in bed, comfortable and in NAD.
ROS: Significant findings: MMM, cardiac murmur, 2(+) pedal edema BL, A&O x 2
VS: Temperature: 98.2°F; heart rate: 90 beats per minute; blood pressure: 140/90 mm Hg; respiratory rate:
18 breaths per minute; O2 saturation: 97% RA
Weight: 129 lb (baseline 119 lb); height: 5 ft, 2 in
Laboratory Values
Na: 130
Cl: 107 Total
Blood urea nitrogen (BUN): 82
BG: 88
Hct: 30.4%
Platelets: 212
K: 6.4
CO2: 18
Serum Cr: 4.0
White blood cell (WBC) count: 10.1
Hgb: 10.0
U/A: C&S pending; SG. 1.015; pH 7.5, Na: 20, Cr: 52, /hpf RBC: 0–2; WBC: 6–10,
Bacteria (occ); yeast (–)
UO: < 10 mL/h
ECG: No QRS widening or T-wave elevations
Chest x-ray: Bilateral pleural effusions; no evidence of pneumonia
Medications administered in emergency room: Regular insulin 10 units and 50% dextrose 50 mL IV x 1,
ciprofloxacin (Cipro) 400 mg IV q12h
Choose the factor that is associated with improved patient outcomes in ARF.
A. Nonoliguria
B. Advanced age
C. Hospitalization
D. Comorbidity
E. All the above
QUESTION 50:
INCORRECT (You answered A; the correct answer is C)
HL is a 69-year-old woman who presents to the emergency room complaining of severe nausea and vomiting for 3
days. On admission her serum creatinine is 2.0 mg/dL (her baseline is 0.8 mg/dL). She has not been able to eat or
drink for 3 days and has lost 2.5 kg. Her medications on admission include: hydrochlorothiazide 25 mg po every day,
lisinopril 10 mg po every day. Which of the following statements isfalse regarding HL at this time?
A. Nausea and vomiting may have caused a decrease in her effective arterial blood volume leading to pre-renal
AKI.
B. HL should not receive radio contrast media unless absolutely necessary until her kidneys recover.
C. Hydrochlorothiazide may have caused vasoconstriction of the afferent arteriole leading to pre-renal AKI.
D. Lisinopril should be discontinued until HL's kidney function returns near her baseline.
E. HL's weight loss suggests fluid volume depletion.
QUESTION 51:
CORRECT (You correctly answered A)
In continuous renal replacement therapy (CRRT), the following statement is true regarding drug clearance:
A. Increasing CRRT ultrafiltration rate will generally result in increased drug clearance.
B. Decreasing the CRRT ultrafiltration rate will generally result in increased drug clearance.
C. Decreasing the CRRT dialysate rate will generally result in increased drug clearance.
D. Increasing the CRRT dialysate rate will generally result in decreased drug clearance.
QUESTION 52:
INCORRECT (You answered C; the correct answer is D)
Presentation
CC: Patient was transported from home to the emergency room in a private ambulance.
HPI: The patient is a 67-year-old man who was discharged from the hospital with resolving pneumonia one
day PTA. He is accompanied by his son, who claims to have found him unconscious on the bathroom floor. On
arrival to the emergency room, the patient remains unconscious and unresponsive to stimuli, with shallow
respiratory breathing.
PMH: Type 2 diabetes mellitus (DM) x 15 years, hypertension (HTN) x 10 years, mild chronic renal
insufficiency (CRI) x 2 years, congestive heart failure (ACC/AHA stage B) x 1 year, asthma (unknown history),
community-acquired pneumonia (recent hospital admission), headaches
Medications on admission: Levofloxacin (Levoquin) 500 mg PO qd x 1 week left, lisinopril (Zestril) 20 mg qd,
furosemide (Lasix) 20 mg qd in the morning, carvedilol (Coreg) 25 mg bid, glyburide (Micronase) 5 mg qd,
albuterol (Proventil) IH 2 puffs qid PRN, ibuprofen (Advil) 200 mg (2 tablets) PRN for headaches
ALL: NKDA
SH: Widower, lives alone, retired
FH: Noncontributory
Physical Examination
General: White male who is semiconscious and ill-appearing with labored breathing.
HEENT: EOMI, PERRLA, MMM
Neck: Soft, supple
COR: RRR, S1, S2 gallop
PULM: Rales lower left lobe
ABD: Soft, nontender, nondistended
EXT: Positive distal pulses, positive BL edema
Neuro: Not able to perform tests
VS: Temperature: 98.7°F; blood pressure: 124/84 mm Hg; heart rate: 80 beats per minute; respiratory rate:
15 breaths per minute
Weight: 58 kg; height: 5 ft, 6 in
Laboratory Values
Na: 140 mEq/L
Cl: 99
Blood urea nitrogen (BUN): 101
Glucose: 160
Red blood cell (RBC) count: 4.2
Hgb: 11.4
Ca: 7.9
Albumin: 4.0
U/A: No sediment, no protein, Gram's stain (–)
Urine output: <10 mL/h
ECG: Flattened P waves, widened QRS segment
K: 7.5
Total CO2: 18
Serum Cr: 6.2 (baseline 1.4)
White blood cell (WBC) count: 15
Hct: 35%
Platelets: 180
PO4: 8.2
The medical team makes the following assessments:
1. ECG changes consistent with life-threatening hyperkalemia
2. Oliguric acute renal failure probably due prerenal azotemia from volume overload
3. r/o septicemia
Which of the following drugs may have exacerbated hyperkalemia in the presence of declining renal function in this
patient?
A. Levofloxacin
B. Carvedilol
C. Furosemide
D. Lisinopril
E. Glyburide
QUESTION 53:
INCORRECT (You answered A; the correct answer is E)
Which of the following statements is true regarding the use of diuretics in patients with oliguric AKI?
A. Diuretics may increase urine output and reverse kidney damage.
B. Diuretics should never be used in AKI as they may cause dehydration and exacerbate AKI.
C. Diuretics can be used in very high doses as there are no toxicities to be concerned with.
D. Thiazides and potassium-sparing diuretics are the preferred agents in AKI.
E. Diuretics may improve urine output and help manage fluid and electrolyte abnormalities.
QUESTION 54:
INCORRECT (You answered E; the correct answer is A)
Presentation
CC: Primary care physician referral
HPI: An 83-year-old woman is referred to the hospital for a renal workup. The patient had complaints of
decreased urine output (UO) and a 10-lb weight gain 1 week PTA according to her daughter. On examination
by the medical housestaff, the patient denies chest pain, palpitations, dyspnea, or abdominal pain.
PMH: Congestive heart failure (CHF), hypertension (HTN), urinary incontinence, gout (no active medications),
mild dementia, moderate renal insufficiency (baseline serum creatinine 1.3)
Medications on admission: Nitroglycerin (10 mg) transdermal 0.4 g/h qam, ramipril (Altace) 2.5 mg qd,
hydrochlorothiazide (Hydrodiuril) 25 mg qd, multivitamin 1 tab qd, oxybutynin (Ditropan) 5 mg qd
ALL: Penicillin
FH: Not available
SH: Patient lives with daughter; no alcohol use; 30 pack-year history of tobacco use
PE: Patient looks younger than stated age, lying in bed, comfortable and in NAD.
ROS: Significant findings: MMM, cardiac murmur, 2(+) pedal edema BL, A&O x 2
VS: Temperature: 98.2°F; heart rate: 90 beats per minute; blood pressure: 140/90 mm Hg; respiratory rate:
18 breaths per minute; O2 saturation: 97% RA
Weight: 129 lb (baseline 119 lb); height: 5 ft, 2 in
Laboratory Values
Na: 130
Cl: 107 Total
Blood urea nitrogen (BUN): 82
BG: 88
Hct: 30.4%
Platelets: 212
K: 6.4
CO2: 18
Serum Cr: 4.0
White blood cell (WBC) count: 10.1
Hgb: 10.0
U/A: C&S pending; SG. 1.015; pH 7.5, Na: 20, Cr: 52, /hpf RBC: 0–2; WBC: 6–10,
Bacteria (occ); yeast (–)
UO: < 10 mL/h
ECG: No QRS widening or T-wave elevations
Chest x-ray: Bilateral pleural effusions; no evidence of pneumonia
Medications administered in emergency room: Regular insulin 10 units and 50% dextrose 50 mL IV x 1,
ciprofloxacin (Cipro) 400 mg IV q12h
Regular insulin has a quick onset of action in the management of hyperkalemia, whereby it reduces serum K+ by
promoting K+ movement from the extracellular to the intracellular space. What other agent would you recommend if
the patient experiences persistent hyperkalemia?
A. Sodium polysterene sulfonate (Kayexalate)
B. Calcium gluconate IV
C. Calcium chloride IV
D. Sodium bicarbonate IV
E. Albuterol IH
QUESTION 55:
INCORRECT (You answered A; the correct answer is D)
Which of the following is the least common type of true acute kidney injury (AKI)?
A. Prerenal AKI
B. Pseudorenal AKI
C. Intrinsic AKI
D. Postrenal AKI
QUESTION 56:
INCORRECT (You answered A; the correct answer is B)
Hyperkalemia may result from AKI and can lead to which one of the following life-threatening complications?
A. Seizures
B. Arrhythmias
C. Hypertension
E. Acidosis e. Encephalopathy
QUESTION 57:
INCORRECT (You answered A; the correct answer is C)
Which of the following markers allows for a significantly earlier diagnosis of AKI compared with serum creatinine?
A. Blood urea nitrogen
B. Glomerular filtration rate
C. Neutrophil gelatinase-associated lipocalin
D. Urinary creatinine levels
QUESTION 58:
INCORRECT (You answered A; the correct answer is C)
Which of the following medications should not be used for prevention of AKI in high-risk individuals?
A. Ascorbic acid
B. Sodium bicarbonate
C. Dopamine
D. N-acetylcysteine
QUESTION 59:
CORRECT (You correctly answered A)
Presentation
CC: Primary care physician referral
HPI: An 83-year-old woman is referred to the hospital for a renal workup. The patient had complaints of
decreased urine output (UO) and a 10-lb weight gain 1 week PTA according to her daughter. On examination
by the medical housestaff, the patient denies chest pain, palpitations, dyspnea, or abdominal pain.
PMH: Congestive heart failure (CHF), hypertension (HTN), urinary incontinence, gout (no active medications),
mild dementia, moderate renal insufficiency (baseline serum creatinine 1.3)
Medications on admission: Nitroglycerin (10 mg) transdermal 0.4 g/h qam, ramipril (Altace) 2.5 mg qd,
hydrochlorothiazide (Hydrodiuril) 25 mg qd, multivitamin 1 tab qd, oxybutynin (Ditropan) 5 mg qd
ALL: Penicillin
FH: Not available
SH: Patient lives with daughter; no alcohol use; 30 pack-year history of tobacco use
PE: Patient looks younger than stated age, lying in bed, comfortable and in NAD.
ROS: Significant findings: MMM, cardiac murmur, 2(+) pedal edema BL, A&O x 2
VS: Temperature: 98.2°F; heart rate: 90 beats per minute; blood pressure: 140/90 mm Hg; respiratory rate:
18 breaths per minute; O2 saturation: 97% RA
Weight: 129 lb (baseline 119 lb); height: 5 ft, 2 in
Laboratory Values
Na: 130
Cl: 107 Total
Blood urea nitrogen (BUN): 82
BG: 88
Hct: 30.4%
Platelets: 212
K: 6.4
CO2: 18
Serum Cr: 4.0
White blood cell (WBC) count: 10.1
Hgb: 10.0
U/A: C&S pending; SG. 1.015; pH 7.5, Na: 20, Cr: 52, /hpf RBC: 0–2; WBC: 6–10,
Bacteria (occ); yeast (–)
UO: < 10 mL/h
ECG: No QRS widening or T-wave elevations
Chest x-ray: Bilateral pleural effusions; no evidence of pneumonia
Medications administered in emergency room: Regular insulin 10 units and 50% dextrose 50 mL IV x 1,
ciprofloxacin (Cipro) 400 mg IV q12h
What are the most clinically significant distinguishing features associated with ARF reported in this case?
A. Elevated serum creatinine, BUN, and K+
B. Accumulation of H+
C. Hyperphosphatemia/hypocalcemia
D. Low Hct and Hgb
E. Urinary sediment
QUESTION 60:
INCORRECT (You answered A; the correct answer is C)
Diuretic resistance to furosemide may be overcome by
A. Switching to a different loop diuretic
B. Increasing the furosemide dose and decreasing the frequency
C. Adding a thiazide diuretic
D. Adding a second loop diuretic
QUESTION 61:
INCORRECT (You answered C; the correct answer is A)
Presentation
CC: Patient was transported from home to the emergency room in a private ambulance.
HPI: The patient is a 67-year-old man who was discharged from the hospital with resolving pneumonia one
day PTA. He is accompanied by his son, who claims to have found him unconscious on the bathroom floor. On
arrival to the emergency room, the patient remains unconscious and unresponsive to stimuli, with shallow
respiratory breathing.
PMH: Type 2 diabetes mellitus (DM) x 15 years, hypertension (HTN) x 10 years, mild chronic renal
insufficiency (CRI) x 2 years, congestive heart failure (ACC/AHA stage B) x 1 year, asthma (unknown history),
community-acquired pneumonia (recent hospital admission), headaches
Medications on admission: Levofloxacin (Levoquin) 500 mg PO qd x 1 week left, lisinopril (Zestril) 20 mg qd,
furosemide (Lasix) 20 mg qd in the morning, carvedilol (Coreg) 25 mg bid, glyburide (Micronase) 5 mg qd,
albuterol (Proventil) IH 2 puffs qid PRN, ibuprofen (Advil) 200 mg (2 tablets) PRN for headaches
ALL: NKDA
SH: Widower, lives alone, retired
FH: Noncontributory
Physical Examination
General: White male who is semiconscious and ill-appearing with labored breathing.
HEENT: EOMI, PERRLA, MMM
Neck: Soft, supple
COR: RRR, S1, S2 gallop
PULM: Rales lower left lobe
ABD: Soft, nontender, nondistended
EXT: Positive distal pulses, positive BL edema
Neuro: Not able to perform tests
VS: Temperature: 98.7°F; blood pressure: 124/84 mm Hg; heart rate: 80 beats per minute; respiratory rate:
15 breaths per minute
Weight: 58 kg; height: 5 ft, 6 in
Laboratory Values
Na: 140 mEq/L
Cl: 99
Blood urea nitrogen (BUN): 101
Glucose: 160
Red blood cell (RBC) count: 4.2
Hgb: 11.4
Ca: 7.9
Albumin: 4.0
U/A: No sediment, no protein, Gram's stain (–)
Urine output: <10 mL/h
ECG: Flattened P waves, widened QRS segment
K: 7.5
Total CO2: 18
Serum Cr: 6.2 (baseline 1.4)
White blood cell (WBC) count: 15
Hct: 35%
Platelets: 180
PO4: 8.2
The medical team makes the following assessments:
1. ECG changes consistent with life-threatening hyperkalemia
2. Oliguric acute renal failure probably due prerenal azotemia from volume overload
3. r/o septicemia
What is the most appropriate agent for the management of hyperkalemia associated with ARF in this patient?
A. Calcium gluconate IV
B. Sodium bicarbonate IV
C. Furosemide IV
D. Regular insulin IV
E. Sodium polysterene sulfonate PO
QUESTION 62:
INCORRECT (You answered A; the correct answer is E)
Which of the following statements is true regarding the nutrition support of patients with acute renal failure?
A. ARF causes significant hypometabolism
B. ARF causes significant hypermetabolism
C. Patients with ARF on dialysis require protein restriction
D. Patients with ARF do not tolerate enteral nutrition
E. ARF does not uniformly alter patient energy needs
QUESTION 63:
CORRECT (You correctly answered A)
According to the Sixth Report of the Joint National Committee on Prevention, Detection, Evaluation, and Treatment of
High Blood Pressure (JNC6) guidelines, patients with renal insufficiency should have a target blood pressure of less
than:
A. 130/80 mm Hg
B. 160/90 mm Hg
C. 140/85 mm Hg
D. 125/75 mm Hg
E. 130/85 mm Hg
QUESTION 64:
INCORRECT (You answered A; the correct answer is D)
Emerging data has shown that the role of angiotensin receptor blockers (ARBs) in the progression of renal disease can
be similar to that of:
A.
-blockers
B.
1-blockers
C. CCBs
D. ACEIs
E. Diuretics
QUESTION 65:
INCORRECT (You answered A; the correct answer is D)
General approaches for prevention of progression of renal disease include:
A. Screening all diabetic patients for microalbuminuria every 2 years
B. Initiating
-blockers in normotensive or hypertensive diabetic patients with persistent microalbuminuria
C. Avoiding use of lipid-lowering agents
D. Maintaining blood glucose within or close to normal range
E. All of the above
QUESTION 66:
INCORRECT (You answered A; the correct answer is C)
For a patient initiating angiotensin-converting enzyme inhibitor (ACEI) therapy, which of the following parameters
should be frequently monitored?
A. Blood pressure and pulse
B. Sodium and potassium
C. Potassium and serum creatinine
D. WBC and proteinuria
E. No monitoring is necessary
QUESTION 67:
INCORRECT (You answered A; the correct answer is C)
Which class of antihypertensive agent has demonstrated both decreased proteinuria and preserved GFR independent
of the degree of blood pressure reduction in diabetic patients?
A. Diuretics
B. CCBs
C. ACEI
D.
-blockers
E.
1-blockers
QUESTION 68:
INCORRECT (You answered A; the correct answer is D)
The most common cause of end-stage renal disease (ESRD) in the United States is:
A. Polycystic kidney disease
B. Glomerulonephritis
C. Hypertension
D. Diabetes mellitus
E. Renal artery stenosis
QUESTION 69:
CORRECT (You correctly answered A)
Which of the following would be the best regimen to initiate in a diabetic, hypertensive patient with stage 4 chronic
kidney disease?
A. ACEI plus loop diuretic
B. ACEI plus thiazide diuretic
C. CCB plus
D. ACEI plus
E.
-blocker
-blocker
-blocker plus loop diuretic
QUESTION 70:
INCORRECT (You answered A; the correct answer is E)
A potential reason for a more rapid progression of renal failure than predicted would be:
A. Poorly controlled hypertension
B. Volume depletion
C. Drug-related effects
D. A and B
E. All of the above
QUESTION 71:
INCORRECT (You answered A; the correct answer is C)
T.A. is a 25-year-old female with type 1 diabetes, hypertension, and a creatinine clearance (CL cr) of 38 mL/min. She
would be classified as having what stage of chronic kidney disease?
A. Stage 1
B. Stage 2
C. Stage 3
D. Stage 4
E. Stage 5
QUESTION 72:
INCORRECT (You answered A; the correct answer is D)
Type 2 diabetic patients should be screened for microalbuminuria every:
A. Month
B. 3 months
C. 6 months
D. 12 months
E. 24 months
QUESTION 73:
INCORRECT (You answered A; the correct answer is C)
In patients with diabetes and a glomerular filtration rate (GFR) <25 mL/min, Kidney Disease/Dialysis Outcome Quality
Initiative (K/DOQI) guidelines on nutrition recommend a dietary protein intake of:
A. 0.20 g/protein/kg/day
B. 0.28 g/protein/kg/day
C. 0.60 g/protein/kg/day
D. 0.75 g/protein/kg/day
E. 1.00 g/protein/kg/day
QUESTION 74:
CORRECT (You correctly answered A)
The National Kidney Foundation Hypertension and Diabetes Executive Committee Working Group recommends that
patients with diabetes have a target blood pressure of less than:
A. 130/80 mm Hg
B. 160/90 mm Hg
C. 140/85 mm Hg
D. 125/75 mm Hg
E. 130/85 mm Hg
QUESTION 75:
INCORRECT (You answered A; the correct answer is B)
Studies demonstrating the beneficial renal effects of more intensive blood glucose control in diabetic patients include:
A. HOPE/DCCT
B. DCCT/UKPDS
C. MRFIT/HOPE
D. UKPDS/ELITE
E. HOT/HOPE
QUESTION 76:
CORRECT (You correctly answered A)
Which class of hypocholesteremic therapies is suggested to be first line according to the K/DOQI guidelines
and may have some advantage over others in patients with renal failure or nephrotic syndrome?
A. HMG-CoA reductase inhibitors
B. Nicotinic acid
C. Bile acid sequestrants
D. Fish oils
E. Fibric acids
QUESTION 77:
INCORRECT (You answered A; the correct answer is B)
Which class of antihypertensive agents have been shown in several studies to decrease the progression of renal
failure in nondiabetic patients?
A.
-blockers
B. ACEI
C. Loop diuretics
D. CCB
E.
1-blockers
QUESTION 78:
INCORRECT (You answered A; the correct answer is D)
ES is a 45-year-old male on HD started on an epoetin dose of 5,000 Units intravenously TIW 1 week ago. The Hb at
the time of initial dosing was 10 g/dL (100 g/L; 6.21 mmol/L). The current Hb is 10.3 g/dL (103 g/L; 6.39 mmol/L).
Iron indices reveal the following: ferritin 250 ng/mL (250
g/L), transferrin saturation 30% (0.30). Which of the
following options is most appropriate for ES?
A. Oral ferrous sulfate 325 mg three times per day
B. 1 g of iron sucrose divided over 10 hemodialysis sessions
C. Increase the dose of epoetin alfa by 50% to achieve the target Hb
D. No change is necessary based on Hb response after 1 week of epoetin alfa
QUESTION 79:
INCORRECT (You answered A; the correct answer is C)
TR is an ESRD patient just starting peritoneal dialysis. The most recent laboratory analysis reveals the following:
phosphorus 7.4 mg/dL (2.39 mmol/L), calcium 9.0 mg/dL (2.25 mmol/L), albumin 2.5 g/dL (25 g/L), iPTH 500 pg/mL
(500 ng/L), 25-OH D 40 ng/mL (100 nmol/L). TR is on sevelamer carbonate as a phosphate binder and no other
therapy to address CKD-MBD. Which of the following is the most appropriate vitamin D therapy?
A. Calcitriol
B. Ergocalciferol
C. Paricalcitol
D. Cholcalciferol
QUESTION 80:
INCORRECT (You answered A; the correct answer is B)
Which of the following regimens is most appropriate for management of severe hyperkalemia in a patient with ESRD
and no residual kidney function?
A. Loop diuretic and dialysis
B. Sodium polystyrene sulfonate and dialysis
C. IV calcium gluconate and loop diuretic
D. Insulin and glucose
QUESTION 81:
INCORRECT (You answered A; the correct answer is D)
In addition to lowering PTH, the calcimimetic agent cinacalcet causes which of the following changes in laboratory
parameters?
A. Increase in phosphorus, decrease in calcium
B. Decrease in phosphorus, increase in calcium
C. Increase in phosphorus and calcium
D. Decrease in phosphorus and calcium
QUESTION 82:
INCORRECT (You answered A; the correct answer is B)
Which of the following secondary complications can usually be effectively treated with chronic dialysis therapy (either
peritoneal dialysis or hemodialysis) alone?
A. Iron deficiency anemia
B. Metabolic acidosis
C. Renal osteodystrophy
D. Hyperlipidemia
QUESTION 83:
CORRECT (You correctly answered A)
Achievement of a higher target hemoglobin level (> 12 g/dL [>120 g/L; 7.45 mmol/L]) in the CKD population has
been associated with which of the following?
A. Higher risk of mortality
B. Improved survival
C. Decrease in quality of life
D. Lower risk of cardiovascular events
QUESTION 84:
INCORRECT (You answered A; the correct answer is C)
A patient with ESRD and the associated secondary complications would likely present with which of the following
laboratory data?
A. Phosphorus 2.0 mg/dL (0.65 mmol/L), Hb 13 g/dL (8.07 mmol/L)
B. PTH 275 pg/mL (275 ng/L), Hb 11 g/dL (110 g/L; 6.83 mmol/L)
C. Phosphorus 6.0 mg/dL (1.94 mmol/L), serum bicarbonate 15 mEq/L (15 mmol/L)
D. PTH 110 pg/mL (110 ng/L), serum bicarbonate 22 mEq/L (22 mmol/L)
QUESTION 85:
CORRECT (You correctly answered A)
Which of the following is the recommended total daily energy intake for a patient with ESRD on chronic hemodialysis?
A. 35 kcal/kg (147 kJ/kg)
B. 25 kcal/kg (105 kJ/kg)
C. 40 kcal/kg (167 kJ/kg)
D. 20 kcal/kg (84 kJ/kg)
QUESTION 86:
CORRECT (You correctly answered A)
Which characteristics define CKD-mineral and bone disorder?
A. Abnormal calcium, phosphorus, PTH and vitamin D levels
B. Presence of anemia, CKD and renal osteodystrophy
C. Low phosphorus and osteoporosis
D. Failure to correct 25-hydroxyvitamin D levels with vitamin D supplementation
QUESTION 87:
INCORRECT (You answered A; the correct answer is C)
According to the K/DOQI guidelines for anemia management which of the following is the preferred route of iron
administration in the nondialysis CKD population?
A. Intravenous
B. Oral
C. Either oral or intravenous
D. Intramuscular
QUESTION 88:
CORRECT (You correctly answered A)
Which of the following iron preparations requires a test dose because of the association with anaphylactic reactions?
A. Iron dextran
B. Sodium ferric gluconate
C. Iron sucrose
D. Ferumoxytol
QUESTION 89:
INCORRECT (You answered A; the correct answer is D
Which of the following is a typical repletion dose of IV iron recommended in the hemodialysis population with absolute
iron deficiency?
A. 100 mg per week for 8 weeks
B. 250 mg administered over 1 hour
C. 500 mg administered in divided doses
D. 1000 mg administered in divided doses
QUESTION 90:
INCORRECT (You answered A; the correct answer is D)
Which of the following is an appropriate statement regarding the epidemic of CKD in the United States?
A. The number of individuals with CKD is expected to decline in the next 10 years due to increased awareness of
CKD.
B. The mortality rate is lower for an individual with ESRD compared with age-matched individuals without CKD.
C. Dialysis therapy corrects the majority of secondary complications of CKD, and these individuals require fewer
treatment interventions compared with early-stage CKD patients
D. The incident rate of ESRD is much higher in African Americans and Hispanics compared with that in whites.
QUESTION 91:
CORRECT (You correctly answered A)
Which of the following phosphate-binding agents would be recommended for an individual who has difficulty
swallowing larger pills?
A. Lanthanum carbonate tablets
B. Aluminum hydroxide liquid
C. Calcium carbonate tablets
D. Sevelamer carbonate tablets
QUESTION 92:
INCORRECT (You answered A; the correct answer is C)
Which of the following diuretic regimens would likely be required to optimize diuresis in an individual with stage 4 CKD
(GFR 20 mL/min/1.73 m2 [0.19 mL/s/m2]) requiring volume removal?
A. Metolazone alone
B. Hydrochlorothiazide + metolazone
C. Furosemide + metolazone
D. Furosemide + spironolactone
QUESTION 93:
INCORRECT (You answered A; the correct answer is E)
Which of the following medications should have the dose adjusted in patients with a decreased renal clearance?
A. Gabapentin
B. Levofloxacin
C. Ranitidine
D. B and C
E. A, B, and C
QUESTION 94:
INCORRECT (You answered A; the correct answer is D)
Which of the following drugs is least likely to be removed by conventional hemodialysis (i.e., hemodialysis using a
cellulose membrane)?
A. Foscarnet (MW = 94; Vd = 0.7 1/kg; Fraction bound = 0.17)
B. Cefazolin (MW = 454; Vd = 0.2 1/kg; Fraction bound = 0.50)
C. Ceftriaxone (MW = 450; Vd = 0.2 1/kg; Fraction bound = 0.90)
D. Inulin (MW = 5,200; Vd = 0.05 1/kg; Fraction bound = 0.01)
QUESTION 95:
INCORRECT (You answered A; the correct answer is E)
T.D. is a 34-year-old, 60-kg man with a CLcr of 8 mL/min who has been receiving hemodialysis for 3 months. He was
just started on gabapentin 300 mg orally every 8 hours by his neurologist. What dosage regimen would you
recommend given that the volume of distribution is 0.7 L/kg, the fraction eliminated in the urine unchanged is 90%,
and protein binding is 3%?
A. 300 mg every 8 hours
B. 150 mg every 8 hours
C. 50 mg every 8 hours
D. 300 mg every 24 hours
E. 300 mg every 48 hours
QUESTION 96:
INCORRECT (You answered A; the correct answer is C)
Which of the following drug(s) will most likely have an increased fraction unbound in patients with end-stage renal
disease (ESRD)?
A. Clonidine
B. Disopyramide
C. Phenytoin
D. Propafenone
E. A and B
QUESTION 97:
CORRECT (You correctly answered A)
P.M. is a 65-year old male diagnosed with stage IV with non-small-cell lung cancer about to receive carboplatin plus
paclitaxel. The order is written as follows: carboplatin AUC 6. The patient has a calculated CrCl of 80 mL/min (1.33
mL/s) and a BSA of 2m2. What dose of carboplatin should PM receive?
A. 630 mg
B. 1,260 mg
C. 6 mg
D. 12 mg
QUESTION 98:
INCORRECT (You answered A; the correct answer is C)
L.Y. is scheduled to receive her first dose of a chemotherapy regimen containing the following agents: doxorubicin,
methotrexate, and vinblastine. Her creatinine clearance is approximately 30 mL/min (0.50 mL/s) [estimated], and her
bilirubin is 2.4 mg/dL (41
mol/L) (normal <1.2 mg/dL [20.5
mol/L]). Which of the following empiric dose
modifications should be recommended for these three drugs, based on L.Y.'s pattern of organ function?
A. Reduce doxorubicin and vinblastine doses by 50%; no change in methotrexate dose.
B. Reduce the methotrexate dose by 50%; no change in doxorubicin and vinblastine doses.
C. Reduce doses of all agents by 50%.
D. No chemotherapy doses should be given.
QUESTION 99:
INCORRECT (You answered A; the correct answer is D)
S.M. is a 78-year-old, 75-kg man with a residual CLcr of 9 mL/min who has been receiving hemodialysis for 9 months.
He received gentamicin 150 mg IV over 0.5 hours to treat a soft tissue infection. What do you project his serum
concentration will be in 40 hours, given that the volume of distribution is 0.3 L/kg, the CLgentamicin = CLcr × 0.983?
A. 8.3 mg/L
B. 6.6 mg/L
C. 4.8 mg/L
D. 2.5 mg/L
E. 1.8 mg/L
QUESTION 100:
CORRECT (You correctly answered A)
If the measured serum gentamicin concentration was 3.6 mg/L, what do you project the gentamicin serum
concentration will be after 4 hours of dialysis with a high-flux dialyzer for which the reported gentamicin clearance is
116 mL/min?
A. 0.7 mg/L
B. 1.1 mg/L
C. 1.6 mg/L
D. 2.0 mg/L
E. 2.5 mg/L
QUESTION 101:
CORRECT (You correctly answered A)
J.S. is a 70-year old African American male (5'8", 85 kg) with a history of hypertension and CKD. His serum creatinine
today is 1.50 mg/dL (using the IDMS calibrated assay). What is J.S.'s estimated GFR when corrected for BSA (in
mL/min)?
A. 38.9 mL/min
B. 45.6 mL/min
C. 65.2 mL/min
D. 72.1 mL/min
QUESTION 102:
CORRECT (You correctly answered A)
Unbound drug concentrations for drugs that are highly protein bound should be used to monitor therapy and make
dose modifications in patients with chronic kidney disease.
A. True
B. False
QUESTION 103:
INCORRECT (You answered A; the correct answer is D)
Two days after admission, the microbiology sputum report identifies a gram-negative organism susceptible to
gentamicin, not penicillin. An appropriate gentamicin order would be
A. 7 mg/kg IV q 24 h
B. 100 mg PO q 8 h
C. 1.5 mg/kg IM q 8 h
D. 1.5 mg/kg IV q 36 h
E. 3 mg IV q 8 h
Related Topics:
- gentamicin sulfate (usp)
QUESTION 104:
INCORRECT (You answered A; the correct answer is D)
Based on Ms. Grant's profile, the penicillin order should be questioned because
A. dose is too low
B. dose is too high
C. patient is hypokalemic
D. patient is hyperkalemic
E. Jarisch-Herxheimer reaction is likely to occur
QUESTION 105:
INCORRECT (You answered A; the correct answer is C)
A.C. is to receive amoxicillin for a suspected urinary tract infection. His measured CL cr is 45 mL/min, and the
clearance and fraction of amoxicillin eliminated renally unchanged in a patient with normal renal function (CLcr = 120
mL/min) are 221 mL/min and 86%, respectively. What do you project his clearance of amoxicillin will be?
A. 150 mL/min
B. 119 mL/min
C. 102 mL/min
D. 83 mL/min
E. 19 mL/min
QUESTION 106:
INCORRECT (You answered A; the correct answer is B)
J.R. is a 68 year-old Caucasian man (60 kg, 5'7") with a history of hypertension, cerebral stroke and benign prostatic
hypertrophy. He presents to the ambulatory care clinic today for evaluation of a viral infection to be treated with
acyclovir. His serum creatinine value today is 0.63 mg/dL. Which one of the following approaches should be used to
assess this patient's renal function for the purpose of renal dose adjustment for acyclovir?
A. Measure a chromium-labeled ethylenediaminetetraacetic acid GFR.
B. Estimate creatinine clearance using the CG equation.
C. Estimate GFR using the MDRD equation.
D. Conduct a timed 24-hour urine collection.
QUESTION 107:
CORRECT (You correctly answered D
Which of the following statements regarding renal drug excretion is true?
A. The P-glycoprotein transport system in the kidney is also involved in the tubular secretion of anionic drugs
B. The decrease in renal drug clearance for drugs that are primarily secreted isproportional to the reduction in
glomerular filtration rate
C. The renal clearance of a drug that is actively secreted is likely to be reduced more in a patient with
glomerulonephritis than in one with tubulointerstitial renal disease
D. A patient with acute tubular necrosis will have a lower renal clearance of ampicillin than a patient with chronic
glomerulonephritis (assuming they have comparable CLcr)
QUESTION 108:
INCORRECT (You answered B; the correct answer is C)
A 72-year-old, 65-kg man with a measured CLcr of 48 mL/min is to receive ciprofloxacin. The usual dose of
ciprofloxacin is 500 mg twice daily for patients with normal renal function. Calculate the dose you would recommend
to be given every 12 hours. The relationship between ciprofloxacin oral clearance (CL/F) and renal function is CL/F
(mL/min) = 2.83 (CLcr) + 363.
A. 125 mg every12 hours
B. 250 mg every 12 hours
C. 375 mg every 12 hours
D. 500 mg every 12 hours
QUESTION 109:
INCORRECT (You answered E; the correct answer is D)
Which of the following regimens is the worst choice for a patient with stage IV NSCLC and preexisting renal
dysfunction?
A. Carboplatin–docetaxel
B. Carboplatin–paclitaxel (3 hour infusion)
C. Gemcitabine-paclitaxel
D. EP
E. Carboplatin-vinorelbine
QUESTION 110:
INCORRECT (You answered E; the correct answer is D)
An HIV-infected patient who has a creatinine clearance (CLcr) of 34 mL/min is to receive an anti retroviral that has
a fe of 75%. Based on this information, calculate the most appropriate dosing interval for this patient. The normal
dosing interval (TN) is every 24 hours.
A. 12 hours
B. 24 hours
C. 36 hours
D. 48 hours
E. 72 hours
QUESTION 111:
INCORRECT (You answered E; the correct answer is B)
Which of the following is the optimal approach to determine the effect of hemodialysis on the pharmacokinetics of a
new drug?
A. Determine the half-life of the drug during dialysis and compare with the value observed in those with normal
renal function
B. Calculate the recovery clearance, CL = amount in dialysate/AUC0-t in serum during dialysis
C. Collect the dialysate and calculate the ratio of the concentration of the drug in dialysate to concentration of
drug in blood
D. Measure the blood clearance, CLblood = Qb[(CLarterial –CLvenous)/CLarterial]
E. Determine the total-body clearance of the drug when administered on a nondialysis day relative to the totalbody clearance observed when the drug is given during dialysis
QUESTION 112:
CORRECT (You correctly answered D)
The metabolism of some drugs is altered in patients with renal insufficiency. Select which of the following statements
is true regarding this phenomenon:
A. The onset of the effect on metabolism is rapid, thus no difference has been noted in patients with AKI or CKD
B. Reductions in nonrenal clearance in CKD patients are generally not proportional to the reductions in glomerular
filtration rate (GFR)
C. The effect on metabolism is greater in patients with AKI than in patients with ESRD
D. Data suggest a differential effect on the individual CYP450 enzymes with the activity of some enzymes being
reduced while others are not affected
QUESTION 113:
INCORRECT (You answered D; the correct answer is B)
Which of the following is the predominant mechanism by which the bioavailability of drugs is increased in patients
with stage 5 chronic kidney disease?
A. Decreased renal clearance
B. Decreased first-pass metabolism
C. Increased volume of distribution
D. Increased plasma protein binding
QUESTION 114:
CORRECT (You correctly answered C)
Which of the following statements concerning the relative efficiency of drug removal by dialysis is false?
A. Peritoneal dialysis is less effective than hemodialysis at removing drugs
B. High-flux hemodialysis drug clearances are greater than the most aggressive CVVHD treatments
C. The clearance (mL/min) values of all drugs with high-flux hemodialysis are less than the values reported with
conventional hemodialysis
D. CVVH clearances of a given drug tend to be less than CVVHDF clearances